3
$\begingroup$

Fix a perfectoid field $K$ in mixed characteristic with ring of integers $\mathcal{O}$ and pseudo-uniformizer $\varpi$. Its tilt is the fraction field of $\mathcal{O}^{\flat}=\varprojlim_{x\mapsto x^{p}}\mathcal{O}/\varpi$. Pick an element $\pi\in\mathcal{O}^{\flat}$ such that $\pi^{\sharp}/p\in\mathcal{O}^{\times}$.

I want to understand Fontaine's infinitesimal period ring $$ A_{\text{inf}}:=W(\mathcal{O}^{\flat}). $$ Many references claim that it is complete with respect to the $(p,[\pi])$-adic topology. However, I was not able to find a reference for this statement by myself. I would be grateful if someone could provide a full proof here.

$\endgroup$
3
  • 2
    $\begingroup$ I don't know the original reference, but it is covered in BMS1 Lemma 3.2. $\endgroup$
    – Z. M
    Dec 26, 2022 at 18:15
  • $\begingroup$ @Z. M I am aware of this Lemma. But I am not sure how exactly it gives an answer to my question. $\endgroup$
    – user141099
    Dec 27, 2022 at 8:18
  • $\begingroup$ Erratum: the above reference to BMS1 does not seem to cover this proposition. $\endgroup$
    – Z. M
    Dec 27, 2022 at 13:49

1 Answer 1

2
$\begingroup$

I am not familiar with perfectoid fields, so hopefully my argument is not circular.

We first remark that $(p,[\pi])$ is a regular sequence in $\newcommand\Ainf{A_{\operatorname{inf}}}\Ainf$, since $p$ is a non-zero-divisor and $\pi$ does not vanish in the integral domain $\Ainf/p=\mathcal O^\flat$. Thus the ring $\Ainf$ is $(p,[\pi])$-adically complete if and only if it is derived $(p,[\pi])$-complete, and since the ring $\Ainf$ is already derived $p$-complete, it suffices to check that it is derived $[\pi]$-complete.

Let $\theta\colon\Ainf\to\mathcal O$ denote the Fontaine's map, whose kernel $I$ is principal. Since the ring $\Ainf$ is derived $I$-complete (and in fact, $I$-adically complete), cf. [Hesselholt–Nikolaus, Topological Cyclic Homology, Prop 1.3.4], thus we are reduced to check that the ring $\mathcal O$ is derived $\theta([\pi])$-complete, but $\theta([\pi])=\pi^\sharp$ by definition, and the result follows from the derived $p$-completeness of $\mathcal O$.

$\endgroup$
3
  • $\begingroup$ Many thanks! Just one question: Why is $A_{\text{inf}}$ $(p,[\pi])$-adically complete if and only if it is derived $(p,[\pi])$-adically complete? $\endgroup$
    – user141099
    Jan 2 at 11:18
  • $\begingroup$ @user141099 Since the sequence is Koszul regular, and an explicit criterion for derived completeness. In general, the derived completeness is the "correct" completeness in non-Noetherian setting. Maybe the classical adic completeness is true for sufficiently general perfectoid ring (not necessarily coming from a perfectoid field) where this is not Koszul regular, but I do not know precise statements. $\endgroup$
    – Z. M
    Jan 2 at 11:56
  • $\begingroup$ Thanks for the clarification! $\endgroup$
    – user141099
    Jan 3 at 17:05

Your Answer

By clicking “Post Your Answer”, you agree to our terms of service and acknowledge that you have read and understand our privacy policy and code of conduct.

Not the answer you're looking for? Browse other questions tagged or ask your own question.